There are far fewer independent bookstores than there were 20 years ago, largely because chain bookstores prospered a...

Meredith on November 1, 2019

A v B

Why is A right and B wrong?

Reply
Create a free account to read and take part in forum discussions.

Already have an account? log in

Annie on November 2, 2019

Hi @Meredith,

This question is asking you to find the assumption buried in the argument. This question is a bit tricky because the premises and conclusion are not set up in the normal pattern. Therefore, it's sometimes helpful to reconfigure the argument. Here's a break down:

Premise: The growth of chain bookstores has led to fewer independent bookstores.
Premise: Fewer independent bookstores has resulted in less variety of books.
Conclusion: The growth of chain bookstores is bad for book consumers.

You're looking for an answer which connects the premises and the conclusion. Because this is an assumption question, you're looking for an answer that must be true for the argument to work.

Answer Choices:

(A) is correct. The argument has not explicitly stated that a variety of books is good for consumers. For the conclusion to be true, you must assume that a variety of books is good for consumers. Therefore, this answer is right.

(B) is incorrect. This answer is tricky as it does support the idea that fewer independent bookstores is bad. However, it does not connect the premises and the conclusion. You're looking for an answer choice that fits the gap exactly. The conclusion is about what is good/bad for book consumers. This answer choice does not directly address that.

(C) is incorrect. This is irrelevant to the lack of variety of books.

(D) is incorrect. Like C, this is irrelevant to the variety of books available.

(E) is incorrect. There is no discussion of prices in the stimulus, and therefore this is irrelevant.